LSAT and Law School Admissions Forum

Get expert LSAT preparation and law school admissions advice from PowerScore Test Preparation.

User avatar
 Dave Killoran
PowerScore Staff
  • PowerScore Staff
  • Posts: 5852
  • Joined: Mar 25, 2011
|
#26542
Complete Question Explanation
(The complete setup for this game can be found here: lsat/viewtopic.php?t=11080)

The correct answer choice is (A)

The condition in the question stem creates the following configuration:
October 05_game#1_M12_L3_explanations_game#2_#2_diagram_1.png
Due to the operation of the last rule, this configuration can only be placed in two positions:
October 05_game#1_M12_L3_explanations_game#2_#2_diagram_2.png
These two partial templates eliminate the four incorrect answer choices:

Answer choice (B) can be eliminated because R must be displayed first or second.

Answer choice (C) can be eliminated because Q must be displayed third or fourth.

Answer choice (D) can be eliminated because Q must be displayed third or fourth.

Answer choice (E) can be eliminated because X or R must be displayed second.

Answer choice (A) is thus proven correct by process of elimination.

Note that in the top template, the other variables can be filled in as well. Because S must be first or last, S must be last, and because W cannot be consecutive with T, W must be sixth, leaving P to be displayed fifth:
October 05_game#1_M12_L3_explanations_game#2_#2_diagram_3.png
You do not have the required permissions to view the files attached to this post.

Get the most out of your LSAT Prep Plus subscription.

Analyze and track your performance with our Testing and Analytics Package.